PEDS Neurological questions, Peds neuro, Ch. 48 Musculoskeletal or Articular Dysfunction, Chapter 49: Neuromuscular or Muscular Dysfunction, Chapter 45: Cerebral Dysfunction

¡Supera tus tareas y exámenes ahora con Quizwiz!

A 5-year-old girl sustained a concussion when she fell out of a tree. In preparation for discharge, nurse is discussing home care with her mother. Which statement made by the mother indicates a correct understanding of the teaching?

"I should expect my child to have some behavioral changes after the accident."If the child has episodes of vomiting, sleep disturbances, or diplopia, they should be immediately reported for evaluation

The adolescent is discharged with a prescription for phenytoin (Dilantin). Which of the following statements by the father indicates that discharge teaching has been effective?

"I will be sure to encourage good oral care and regular dental visits."

A child who has a concussion is being discharged home with her parents. Which of the following statements by the parent indicates a need for further teaching?

"I will keep him awake all night to make sure there is no change."

The nurse is providing discharge instructions for a child who has suffered a head injury within the last four hours. The nurse determines there is a need for additional teaching when the mother states:

"My child should sleep for at least 8 hours without arousing after we get home."

The nurse overhears a group of student nurses in the break room discussing the role of the health care professional in suspected child abuse. The nurse concludes that the student who most accurately understands the role is one who makes which of the following statements?"

"Nurses are required to report any case of suspected child abuse to child protective services."

The nurse and a new nurse are caring for a child who will require palliative care. Which of the following statements made by the new nurse would indicate a correct understanding of palliative care?

"Palliative care provides pain and symptom management for the child."

The parent of a child with neuroblastoma verbalizes regret at not coming in earlier for the child's complaints. An appropriate response is:

"This is a silent tumor, which is difficult to diagnose early."

A nurse is caring for a child who has Cerebral palsy. Which of the following medications should the nurse expect to administer to treat painful muscle spasma? SATA

1. Baclofen 2. Diazepam 3. Oxybutynin 4. Methotrexate 5. Prednisone Answer: 1, 2

The pediatric unit charge nurse is working with a newly graduated RN who has been on orientation in the unit for 2 months. Which patient should the charge nurse assign to the new RN?

1. a 2 year old patient with a ventricular septal defect for whom digoxin 90 mcg by mouth has been prescribed 2. A 4 year old patient who had a pulmonary artery banding and has just been transferred in from the ICU 3. a 9 year old with mitral valve endocarditis whose parents need teaching about IV antibiotic administration 4. A 16 year old patient with a heart transplant who was admitted with a low grade fever and tachycardia Answer: 1

After getting change of shift report, which patient should the nurse assess first?

1. an 18 month old patient with coarctation of the aorta who has decreased pedal pulses 2. a 3 year old patient with rheumatic fever who reports severe knee pain 3. a 5 year old patient with endocarditis who has crackles audible throughout both lungs 4. an 8 year old patient with Kawasaki disease who has a temp of 102.2 Answer: 3

A nurse is caring for an infant who has a myelomeningocele. Which of the following actions should the nurse include in the preoperative plan of care?

1. assist the mother with cuddling the infant 2. assess the infants temp rectally 3. place the infant in a supine postition 4. apply a sterile, moist, dressing on the sac Answer: 4

The nurse is caring for a newborn with a myelomeningocele who is awaiting surgical closure of the defect. Which assessment finding is of most concern?

1. bulging of the sac when the infant cries 2. oozing of stool from the anal sphincter 3. flaccid paralysis of both legs 4. temperature of 101.8 Answer: 4

A nurse is caring for a school age child who has juvenile idiopathic arthritis. which of the following home care instructions should the nurse include in the teaching? SATA

1. provide extra time for completing ADLs 2. Use cold compresses for joint pain 3. take ibuprofen on an empty stomach 4. remain home during periods of exacerbation 5. perform ROM exercises Answer: 1, 5

A nurse is caring for a child who has muscular dystrophy. for which of the following findings should the nurse assess? SATA

1. purposeless, involuntary, abnormal movements 2. spinal defect and sac-like protrusion 3. muscular weakness in lower extremities 4. unsteady, wide based or waddling gait 5. upward slant to the eyes Answer: 3, 4

A nurse is developing a plan of care for a toddler who has cerebral palsy. Which of the following actions should the nurse include?

1. structure interventions according to the toddler's chronological age 2. Evaluate the toddler's need for an evaluation of hearing ability 3. monitor the toddlers pain level routinely using a numeric rating scale 4. provide total care for daily hygiene activities Answer: 2

The nurse obtains this information when assessing a 3 year old patient with uncorrected tetralogy of fallot who is crying. Which findings requires immediate action?

1. the apical pulse rate is 118 2. a loud systolic murmur is heard in the pulmonic area 3. there is marked clubbing of the child nail beds 4 the lips and oral mucosa are dusky in color Answer: 4

The nurse is obtaining the history and physical information for a child who is recovering from Kawasaki disease and receives aspirin therapy. Which information concerns the nurse the most?

1. the child attends a day care center 5 days a week 2. the childs fingers have areas of peeling skin 3. the child is very irritable and cries frequently 4. the child has not received any immunization Answer: 4

A 6 year old girl arrives at the ER with her parents. She hit her head when she fell from the jungle gym in the school playground. Which questions are appropriate fro the nurse to ask to assess the child's neurologic status? SATA

1. what is your home address 2. what time does your family eat dinner 3. what grade are you in 4. what is your teachers name 5. what time did you fall 6. what is the name of your school Answer: 1, 3, 4, 6

The nurse notes that an infant with the diagnosis of hydrocephalus has a head that is heavier than that of the average infant. The nurse should determine that special safety precautions are needed when moving the infant with hydrocephalus. Which statement should the nurse plan to include in the discharge teaching with the parents to reflect this safety need?

1."Feed your infant in a side-lying position." 2."Place a helmet on your infant when in bed." 3."Hyperextend your infant's head with a rolled blanket under the neck area." 4."When picking up your infant, support the infant's neck and head with the open palm of your hand." Answer: 4

A nursing student is assisting a school nurse in performing scoliosis screening on the children in the school. The nurse assesses the student's preparation for conducting the screening. The nurse determines that the student demonstrates understanding of the disorder when the student states that scoliosis is characterized by which finding?

1.Abnormal lateral curvature of the spine 2.Abnormal anterior curvature of the lumbar spine 3.Excessive posterior curvature of the thoracic spine 4.Abnormal curvature of the spine caused by inflammation Answer: 1

A child is admitted to the hospital with a diagnosis of acute bacterial meningitis. In reviewing the health care provider's prescriptions, which would the nurse question as appropriate for a child with this diagnosis?

1.Administer an oral antibiotic. 2.Maintain strict intake and output. 3.Draw blood for a culture and sensitivity. 4.Place the child on droplet precautions in a private room. Answer: 1 Administered IV route, not orally

The parents of a child recently diagnosed with cerebral palsy ask the nurse about the limitations of the disorder. The nurse responds by explaining that the limitations occur as a result of which pathophysiological process?

1.An infectious disease of the central nervous system 2.An inflammation of the brain as a result of a viral illness 3.A chronic disability characterized by impaired muscle movement and posture 4.A congenital condition that results in moderate to severe intellectual disabilities Answer: 3

The nurse is assessing a child with increased intracranial pressure. On assessment, the nurse notes that the child is now exhibiting decerebrate posturing. The nurse should modify the client's plan of care based on which interpretation of the client's change?

1.An insignificant finding 2.An improvement in condition 3.Decreasing intracranial pressure 4.Deteriorating neurological function Answer: 4

A child is diagnosed with Reye's syndrome. The nurse creates a nursing care plan for the child and should include which intervention in the plan?

1.Assessing hearing loss 2.Monitoring urine output 3.Changing body position every 2 hours 4.Providing a quiet atmosphere with dimmed lighting Answer: 4

The nurse is creating a plan of care for a newborn infant with spina bifida (myelomeningocele type). The nurse includes assessment measures in the plan to monitor for increased intracranial pressure. Which assessment technique should be performed that will best detect the presence of an increase in intracranial pressure?

1.Check urine for specific gravity. 2.Monitor for signs of dehydration. 3.Assess anterior fontanel for bulging. 4.Assess blood pressure for signs of hypotension. Answer: 3

A lumbar puncture is performed on a child suspected to have bacterial meningitis, and cerebrospinal fluid (CSF) is obtained for analysis. The nurse reviews the results of the CSF analysis and determines that which results would verify the diagnosis?

1.Clear CSF, decreased pressure, and elevated protein level 2.Clear CSF, elevated protein, and decreased glucose levels 3.Cloudy CSF, elevated protein, and decreased glucose levels 4.Cloudy CSF, decreased protein, and decreased glucose levels Answer: 3

The nurse is caring for an infant with spina bifida (myelomeningocele type) who had the sac on the back containing cerebrospinal fluid, the meninges, and the nerves (gibbus) surgically removed. The nursing plan of care for the postoperative period should include which action to maintain the infant's safety?

1.Covering the back dressing with a binder 2.Placing the infant in a head-down position 3.Strapping the infant in a baby seat sitting up 4.Elevating the head with the infant in the prone position Answer: 4

The nurse receives a telephone call from the admissions office and is told that a child with acute bacterial meningitis will be admitted to the pediatric unit. The nurse prepares for the child's arrival and plans to implement which type of precautions?

1.Enteric 2.Contact 3.Droplet 4. Neutropenic Answer: 3

The nurse is providing instructions to the parents of an infant with a ventriculoperitoneal shunt. The nurse should include which instruction?

1.Expect an increased urine output from the shunt. 2.Notify the health care provider if the infant is fussy. 3.Call the health care provider if the infant has a high-pitched cry. 4.Position the infant on the side of the shunt when the infant is put to bed. Answer: 3

The nurse is assigned to care for a child with a brain injury who has a temporal lobe herniation and increasing intracranial pressure. Which signs should the nurse identify as indicative of this type of injury? Select all that apply.

1.Flaccid paralysis 2.Pupil response to light 3.Ipsilateral pupil dilation 4.Compression of the sixth cranial nerve 5.Shifting of the temporal lobe laterally across the tentorial notch Answer: 1, 3, 5 Temporal lobe herniation or uncal herniation refers to a shifting of the temporal lobe laterally across the tentorial notch. This produces compression of the third cranial nerve and ipsilateral pupil dilation. If pressure continues to rise, flaccid paralysis, pupil fixation, and death will result.

The nurse is reviewing the record of a child with increased intracranial pressure and notes that the child has exhibited signs of decerebrate posturing. On assessment of the child, the nurse expects to note which characteristic of this type of posturing?

1.Flaccid paralysis of all extremities 2.Adduction of the arms at the shoulders 3.Rigid extension and pronation of the arms and legs 4.Abnormal flexion of the upper extremities and extension and adduction of the lower extremities Answer: 3

The nurse is reviewing the record of a child with increased intracranial pressure and notes that the child has exhibited signs of decerebrate posturing. Which assessment finding should the nurse expect if this type of posturing is present?

1.Flexion of the upper extremities and extension of the lower extremities. 2.Unilateral or bilateral postural change in which the extremities are rigid. 3.Abnormal extension of the upper and lower extremities with some internal rotation. 4.Arms are adducted with fists clenched, and the legs are flaccid with external rotation. Answer: 3

The nurse is monitoring an infant for signs of increased intracranial pressure. On assessment of the fontanelles, the nurse notes that the anterior fontanelle bulges when the infant is sleeping. Based on this finding, which is the priority nursing action?

1.Increase oral fluids. 2.Document the finding. 3.Notify the health care provider. 4.Place the infant supine in a side-lying position. Answer: 3

The nurse is providing home care instructions to the mother of a child who is recovering from Reye's syndrome. Which instruction should the nurse provide to the mother?

1.Increase stimuli in the home environment. 2.Avoid daytime naps so that the child will sleep at night. 3.Give the child frequent small meals, if vomiting occurs. 4.Check the skin and eyes every day for a yellow discoloration. Answer: 4 Checking for jaundice will assist in identifying the presence of liver complications, which are characteristic of Reye's syndrome. Decreasing stimuli and providing rest decrease stress on the brain tissue. If vomiting occurs in Reye's syndrome, it is caused by cerebral edema and is a sign of intracranial pressure.

The nurse caring for a child who has sustained a head injury in an automobile crash is monitoring the child for signs of increased intracranial pressure (ICP). For which early sign of increased ICP should the nurse monitor?

1.Increased systolic blood pressure 2.Abnormal posturing of extremities 3.Significant widening pulse pressure 4.Changes in level of consciousness Answer: 4

The nurse is caring for a newborn infant with spina bifida (myelomeningocele) who is scheduled for surgical closure of the sac. In the preoperative period, which is the priority problem?

1.Infection 2.Choking 3.Inability to tolerate stimulation 4.Delayed growth and development Answer: 1

The nurse assists a health care provider in performing a lumbar puncture on a 3-year-old child with leukemia in whom central nervous system disease is suspected. In which position will the nurse place the child during this procedure?

1.Lithotomy position 2.Modified Sims' position 3.Lateral recumbent position with the knees flexed and chin resting on the chest 4.Prone with knees flexed to the abdomen and head bent with chin resting on the chest Answer: 3

The nurse is planning care for a child with acute bacterial meningitis. Based on the mode of transmission of this infection, which precautionary intervention should be included in the plan of care?

1.Maintain enteric precautions. 2.Maintain neutropenic precautions. 3.No precautions are required as long as antibiotics have been started. 4.Maintain respiratory isolation precautions for at least 24 hours after the initiation of antibiotics. Answer: 4

The nurse notes documentation that a child is exhibiting an inability to flex the leg when the thigh is flexed anteriorly at the hip. Which condition does the nurse suspect? 1.

1.Meningitis 2.Spinal cord injury 3.Intracranial bleeding 4.Decreased cerebral blood flow Answer: 1

The nurse creates a plan of care for a child with Reye's syndrome. Which priority intervention should the nurse include in the plan of care?

1.Monitor for signs of increased intracranial pressure. 2.Immediately check the presence of protein in the urine. 3.Reassure the parents hyperglycemia is a common symptom. 4.Teach the parents signs and symptoms of a bacterial infection. Answer: 1

A mother arrives at the emergency department with her 5-year-old child and states that the child fell off a bunk bed. A head injury is suspected. The nurse checks the child's airway status and assesses the child for early and late signs of increased intracranial pressure (ICP). Which is a late sign of increased ICP?

1.Nausea 2.Irritability 3.Headache 4.Bradycardia Answer: 4

The nurse is monitoring a 7-year-old child who sustained a head injury in a motor vehicle crash for signs of increased intracranial pressure (ICP). The nurse should assess the child frequently for which early sign of increased ICP?

1.Nausea 2.Papilledema 3.Decerebrate posturing 4.Alterations in pupil size Answer: 1

The community health nurse is providing information to parents of children in a local school regarding the signs of meningitis. The nurse informs the parents that the classic signs/symptoms of meningitis include which findings?

1.Nausea, delirium, and fever 2.Severe headache and back pain 3.Photophobia, fever, and confusion 4.Severe headache, fever, and a change in the level of consciousness Answer: 4

The nurse is reviewing a chart for a child with a head injury. The nurse notes that the level of consciousness has been documented as obtunded. Which finding should the nurse expect to note on assessment of the child?

1.Not easily arousable and limited interaction 2.Loss of the ability to think clearly and rapidly 3.Loss of the ability to recognize place or person 4.Awake, alert, interacting with the environment Answer: 1

The nurse is assigned to care for an 8-year-old child with a diagnosis of a basilar skull fracture. The nurse reviews the health care provider's (HCP's) prescriptions and should contact the HCP to question which prescription?

1.Obtain daily weight. 2.Provide clear liquid intake. 3.Nasotracheal suction as needed. 4.Maintain a patent intravenous line. Answer: 3

The nurse is caring for a child who sustained a head injury after falling from a tree. On assessment of the child, the nurse notes the presence of a watery discharge from the child's nose. The nurse should immediately test the discharge for the presence of which substance?

1.Protein 2.Glucose 3.Neutrophils 4.White blood cells Answer: 2

The nurse caring for an infant with a diagnosis of hydrocephalus should monitor the infant for which sign of increased intracranial pressure?

1.Proteinuria 2.Bradycardia 3.A drop in blood pressure 4.A bulging anterior fontanel Answer: 4

The nurse is assessing for Kernig's sign in a child with a suspected diagnosis of meningitis. Which action should the nurse perform for this test?

1.Tap the child's facial nerve and assess for spasm. 2.Compress the child's upper arm and assess for tetany. 3.Bend the child's head toward the knees and hips and assess for pain. 4.Raise the child's leg with the knee flexed and then extend the leg at the knee and assess for pain. Answer: 4

An infant with a diagnosis of hydrocephalus is scheduled for surgery. Which is the priority nursing intervention in the preoperative period?

1.Test the urine for protein. 2.Reposition the infant frequently. 3.Provide a stimulating environment. 4.Assess blood pressure every 15 minutes. Answer: 2

Cerebral palsy (CP) is suspected in a child and the parents ask the nurse about the potential warning signs of CP. The nurse should provide which information? Select all that apply.

1.The infant's arms or legs are stiff or rigid. 2.A high risk factor for CP is very low birth weight. 3.By 8 months of age, the infant can sit without support. 4.The infant has strong head control but a limp body posture. 5.The infant has feeding difficulties, such as poor sucking and swallowing. 6.If the infant is able to crawl, only one side is used to propel himself or herself. Answer: 1, 2, 5, 6 Cerebral palsy (CP) is a term applied to a disorder that impairs movement and posture. The effects on perception, language, and intellect are determined by the type that is diagnosed. Stiff, rigid arms and legs, low birth weight, poor sucking and swallowing, and inability to crawl properly are potential warning signs of CP. By 8 months of age, if the infant cannot sit up without support, this would be considered a potential warning sign, because this developmental task should be completed by this time. The infant with a potential diagnosis of CP has poor head control by 3 months of age, when head control should be strong.

The nurse is monitoring a child with a brain tumor for complications associated with increased intracranial pressure. Which finding, if noted by the nurse, would indicate the presence of diabetes insipidus?

1.Weight gain 2.Hypertension 3.High urine output 4.Urine specific gravity greater than 1.030 Answer: 3

A child has sustained a fall from a window. Her father finds her completely unconscious and not speaking or moving.What should her Glasgow Coma Scale score be?

3

At what age do most children have an adult concept of death as being inevitable?

9 to 11 years

What effect does immobilization have on the cardiovascular system? a. Venous stasis b. Increased vasopressor mechanism c. Normal distribution of blood volume d. Increased efficiency of orthostatic neurovascular reflexes

A Because of decreased muscle contraction, the physiologic effects of immobilization include venous stasis. This can lead to pulmonary emboli or thrombi. A decreased vasopressor mechanism results in orthostatic hypotension, syncope, hypotension, decreased cerebral blood flow, and tachycardia. An altered distribution of blood volume is found, with decreased cardiac workload and exercise tolerance. Immobilization causes a decreased efficiency of orthostatic neurovascular reflexes, with an inability to adapt readily to the upright position and pooling of blood in the extremities in the upright position.

Which condition can result from the bone demineralization associated with immobility? a. Osteoporosis c. Pooling of blood b. Urinary retention d. Susceptibility to infection

A Bone demineralization leads to a negative calcium balance, osteoporosis, pathologic fractures, extraosseous bone formation, and renal calculi. Urinary retention is secondary to the effect of immobilization on the urinary tract. Pooling of blood is a result of the cardiovascular effects of immobilization. Susceptibility to infection can result from the effects of immobilization on the respiratory and renal systems.

When assessing the child with osteogenesis imperfecta, the nurse should expect to observe: a. Discolored teeth. c. Increased muscle tone. b. Below-normal intelligence. d. Above-average stature.

A Children with osteogenesis imperfecta have incomplete development of bones, teeth, ligaments, and sclerae. Teeth are discolored because of abnormal enamel. Despite their appearance, children with osteogenesis imperfecta have normal or above-normal intelligence. The child with osteogenesis imperfecta has weak muscles and decreased muscle tone. Because of compression fractures of the spine, the child appears short.

An adolescent is scheduled for a leg amputation in 2 days for treatment of osteosarcoma. The nurse's approach should include: a. Answering questions with straightforward honesty. b. Avoiding discussing the seriousness of the condition. c. Explaining that, although the amputation is difficult, it will cure the cancer. d. Assisting the adolescent in accepting the amputation as better than a long course of chemotherapy.

A Honesty is essential to gain the cooperation and trust of the child. The diagnosis of cancer should not be disguised with falsehoods. The adolescent should be prepared in advance for the surgery so that there is time for reflection about the diagnosis and subsequent treatment. This allows questions to be answered. To accept the need for radical surgery, the child must be aware of the lack of alternatives for treatment. Amputation is necessary, but it will not guarantee a cure. Chemotherapy is an integral part of the therapy with surgery. The child should be informed of the need for chemotherapy and its side effects before surgery.

Which term is used to describe a type of fracture that does not produce a break in the skin? a. Simple c. Complicated b. Compound d. Comminuted

A If a fracture does not produce a break in the skin, it is called a simple or closed fracture. A compound or open fracture is one with an open wound through which the bone protrudes. A complicated fracture is one in which the bone fragments damage other organs or tissues. A comminuted fracture occurs when small fragments of bone are broken from the fractured shaft and lie in the surrounding tissue. These are rare in children.

The nurse is preparing an adolescent with scoliosis for a Luque-rod segmental spinal instrumentation procedure. Which consideration should the nurse include? a. Nasogastric intubation and urinary catheter may be required. b. Ambulation will not be allowed for up to 3 months. c. Surgery eliminates the need for casting and bracing. d. Discomfort can be controlled with nonpharmacologic methods.

A Luque-rod segmental spinal instrumentation is a surgical procedure. Nasogastric intubation and urinary catheterization may be required. Ambulation is allowed as soon as possible. Depending on the instrumentation used, most patients walk by the second or third postoperative day. Casting and bracing are required postoperatively. The child usually has considerable pain for the first few days after surgery. Intravenous opioids should be administered on a regular basis.

Osteosarcoma is the most common bone cancer in children. Where are most of the primary tumor sites? a. Femur c. Pelvis b. Humerus d. Tibia

A Osteosarcoma is the most frequently encountered malignant bone cancer in children. The peak incidence is between ages 10 and 25 years. More than half occur in the femur. After the femur, most of the remaining sites are the humerus, tibia, pelvis, jaw, and phalanges.

A child is upset because, when the cast is removed from her leg, the skin surface is caked with desquamated skin and sebaceous secretions. What should the nurse suggest to remove this material? a. Soak in a bathtub. c. Apply powder to absorb material. b. Vigorously scrub the leg. d. Carefully pick material off of the leg.

A Simple soaking in the bathtub is usually sufficient for the removal of the desquamated skin and sebaceous secretions. It may take several days to eliminate the accumulation completely. The parents and child should be advised not to scrub the leg vigorously or forcibly remove this material because it may cause excoriation and bleeding. Oil or lotion, but not powder, may provide comfort for the child.

A young girl has just injured her ankle at school. In addition to calling the child's parents, the most appropriate immediate action by the school nurse is to: a. Apply ice. b. Observe for edema and discoloration. c. Encourage child to assume a comfortable position. d. Obtain parental permission for administration of acetaminophen or aspirin.

A Soft-tissue injuries should be iced immediately. In addition to ice, the extremity should be rested, be elevated, and have compression applied. Observing for edema and discoloration, encouraging the child to assume a comfortable position, and obtaining parental permission or administration of acetaminophen or aspirin are not immediate priorities

Four-year-old David is placed in Buck's extension traction for Legg-Calvé-Perthes disease. He is crying with pain as the nurse assesses that the skin of his right foot is pale with an absence of pulse. What should the nurse do first? a. Notify the practitioner of the changes noted. b. Give the child medication to relieve the pain. c. Reposition the child and notify the physician. d. Chart the observations and check the extremity again in 15 minutes.

A The absence of a pulse and change in color of the foot must be reported immediately for evaluation by the practitioner. Pain medication should be given after the practitioner is notified. This is an emergency condition; immediate reporting is indicated. The findings should be documented with ongoing assessment.

What is most descriptive of the therapeutic management of osteosarcoma? a. Treatment usually consists of surgery and chemotherapy. b. Amputation of the affected extremity is rarely necessary. c. Intensive irradiation is the primary treatment. d. Bone marrow transplantation offers the best chance of long-term survival.

A The optimal therapy for osteosarcoma is a combination of surgery and chemotherapy. Amputation is frequently required. Intensive irradiation and bone marrow transplantation are usually not part of the therapeutic management.

Discharge planning for the child with juvenile arthritis includes the need for: a. Routine ophthalmologic examinations to assess for visual problems. b. A low-calorie diet to decrease or control weight in the less mobile child. c. Avoiding the use of aspirin to decrease gastric irritation. d. Immobilizing the painful joints, which are the result of the inflammatory process.

A The systemic effects of juvenile arthritis can result in visual problems, making routine eye examinations important. Children with juvenile arthritis do not have problems with increased weight and often are anorexic and in need of high-calorie diets. Children with arthritis are often treated with aspirin. Children with arthritis are able to immobilize their own joints. Range-of-motion exercises are important for maintaining joint flexibility and preventing restricted movement in the affected joints.

Often comatose patients cannot cope with the quantity of fluids that they normally tolerate. Overhydration must be avoided to prevent fatal cerebral edema

A 10-year-old boy has been hit by a car while riding his bicycle in front of the school. school nurse immediately assesses airway, breathing, and circulation. The next nursing action should be to: Stabilize neck and spine.

Late signs of ICP in an infant or child include bradycardia, alteration in pupil size and reactivity, decreased motor response, extension or flexion posturing, and Cheyne-Stokes respirations.

A 6-year-old child is having a generalized seizure in the classroom at school. Place in order the interventions the school nurse should implement

What is Munchausen Syndrome by Proxy?

A caregiver who fabricates or induces illness on a child in order to gain control over him or her or because they crave the attention it brings.

Current therapy for a rabid animal bite consists of a thorough cleansing of the wound and passive immunization with human rabies immune globulin as soon as possible.

A child is brought to the emergency department after experiencing a seizure at school. There is no previous history of seizures. The father tells the nurse that he cannot believe the child has epilepsy.

After determining that the child is breathing and has adequate circulation, the next action is to stabilize the neck and spine to prevent any additional trauma

A child is unconscious after a motor vehicle accident. The watery discharge from the nose tests positive for glucose. The nurse should recognize that this suggests: Leaking of cerebrospinal fluid

An infant with hydrocephalus is hospitalized for surgical placement of a ventriculoperitoneal shunt. Observe closely for signs of infection. Maintain an accurate record of intake and output. Monitor for abdominal distention.

A nurse should expect which cerebrospinal fluid laboratory results on a child diagnosed with bacterial meningitis Decreased glucose Cloudy or milky in color Elevated white blood cell count elevated protein

A pediatric client who is known to have cancer is being admitted for mild neutropenia and a severe oral monilial infection. The nurse should assign the child to which room?

A private room without further precautions

During rounds, the interdisciplinary team is discussing a child with leukemia who has just been diagnosed as terminal. The nurses describe the mother's behavior as angry, claiming the nurses are not providing care for her child. The team leader then focuses on which of the following, which is most likely the cause of the mother's anger?

A stage of bereavement over the anticipated loss of the child

Pupils are dilated and fixed. What type of head injury should the nurse suspect? Brainstem

A toddler fell out of a second-story window. She had brief loss of consciousness and vomited four times. Since admission, she has been alert and oriented. Her mother

Accurate and frequent monitoring of intake and output is essential for adjusting fluid volumes to prevent both dehydration and cerebral edema.

A young child's parents call the nurse after their child was bitten by a raccoon in the woods. The nurse's recommendation should be based on knowing that: Antirabies prophylaxis must be initiated.

The nurse is caring for an infant with developmental dysplasia of the hip. Which clinical manifestations should the nurse expect to observe (Select all that apply)? a. Positive Ortolani sign b. Unequal gluteal folds c. Negative Babinski's sign d. Trendelenburg's sign e. Telescoping of the affected limb f. Lordosis

A, B A positive Ortolani sign and unequal gluteal folds are clinical manifestations of developmental dysplasia of the hip seen from birth to 2 to 3 months. Negative Babinski's sign, Trendelenburg's sign, telescoping of the affected limb, and lordosis are not clinical manifestations of developmental dysplasia of the hip.

A school-age child is diagnosed with systemic lupus erythematosus (SLE). The nurse should plan to implement which interventions for this child (Select all that apply)? a. Instructions to avoid exposure to sunlight b. Teaching about body changes associated with SLE c. Preparation for home schooling d. Restricted activity

A, B Key issues for a child with SLE include therapy compliance; body-image problems associated with rash, hair loss, and steroid therapy; school attendance; vocational activities; social relationships; sexual activity; and pregnancy. Specific instructions for avoiding exposure to the sun and ultraviolet B light, such as using sunscreens, wearing sun-resistant clothing, and altering outdoor activities, must be provided with great sensitivity to ensure compliance while minimizing the associated feeling of being different from peers. The child should continue school attendance in order to gain interaction with peers and activity should not be restricted, but promoted.

A 7-month-old male infant diagnosed with spastic cerebral palsy is seen by the nurse in the clinic. Which statement by the parent warrants immediate intervention by the nurse?

A. "My son often chokes while I am feeding him." B. "Is it normal for my child's legs to cross each other?" C. "He gets stiff when I pull him up to a sitting position." D. "My 4-year-old son is jealous of his little brother." Answer: A Airway obstruction is always a priority when caring for any client. Options B and C are characteristics of spastic cerebral palsy and may involve one or both sides. These children have difficulty with fine motor skills, and attempts at motion increase abnormal postures. Option D is an expected behavior and may need to be addressed, but it is not a priority over choking.

The nurse is examining a male child experiencing an exacerbation of juvenile rheumatoid arthritis (JRA) and notes that his mobility is greatly reduced. What is the most likely cause of the child's impaired mobility?

A. Pathologic fractures B. Poor alignment of joints C. Dyspnea on exertion D. Joint inflammation Answer: D Joint inflammation and pain are the typical manifestations of an exacerbation of JRA. Options A, B, and C are not specifically related to JRA.

The nurse assigns an unlicensed assistive personnel (UAP) to provide morning care to a newly admitted child with bacterial meningitis. What is the most important instruction for the nurse to review with the UAP?

A. Use designated isolation precautions. B. Keep the lighting in the room dim. C. Allow the parents to assist with care. D. Report any pain that the child experiences. Answer: A All these are important measures to review with the UAP, but the most important is option A. Improper use of isolation precautions can place other staff and clients at risk for infection. Options B, C, and D promote client comfort and reduce anxiety but are of a lower priority than option A.

A toddler is admitted to the hospital with a possible diagnosis of tetanus. The health care provider has prescribed lorazepam (Ativan) intravenously 0.05 mg/kg/dose every 6 hours prn as a muscle relaxant. The child weighs 22 lbs. How many milligrams of Ativan should the nurse administer per dose? _____ Record your answer using one decimal place.

ANS: 0.5 Find the child's weight in kilograms by dividing 22 by 2.2 = 22/2.2 = 10 kg. Multiply the 0.05-mg dose by 10 = 0.05 mg × 10 kg = 0.5 mg per dose. ch49 Neuromuscular/muscular dysfunction p1593

Latex allergy is suspected in a child with spina bifida. Appropriate nursing interventions include: a. Avoiding using any latex product. b. Using only nonallergenic latex products. c. Administering medication for long-term desensitization. d. Teaching the family about long-term management of asthma.

ANS: A Care must be taken that individuals who are at high risk for latex allergies do not come in direct or secondary contact with products or equipment containing latex at any time during medical treatment. There are no nonallergenic latex products. At this time desensitization is not an option. The child does not have asthma. The parents must be taught about allergy and the risk of anaphylaxis. p1584

Which problem is most often associated with myelomeningocele? a. Hydrocephalus c. Biliary atresia b. Craniosynotosis d. Esophageal atresia

ANS: A Hydrocephalus is an associated anomaly in 80% to 90% of children. Craniosynostosis is the premature closing of the cranial sutures and is not associated with myelomeningocele. Biliary and esophageal atresias are not associated with myelomeningocele. p1580

Spastic cerebral palsy is characterized by: a. Hypertonicity and poor control of posture, balance, and coordinated motion. b. Athetosis and dystonic movements. c. Wide-based gait and poor performance of rapid, repetitive movements. d. Tremors and lack of active movement.

ANS: A Hypertonicity and poor control of posture, balance, and coordinated motion are part of the classification of spastic cerebral palsy. Athetosis and dystonic movements are part of the classification of dyskinetic/athetoid cerebral palsy. Wide-based gait and poor performance of rapid, repetitive movements are part of the classification of ataxic cerebral palsy. Tremors and lack of active movement may indicate other neurologic disorders. ch49 Neuromuscular/muscular dysfunction p1572

The most common problem of children born with a myelomeningocele is: a.Neurogenic bladder. b. Intellectual impairment. c. Respiratory compromise. d. Cranioschisis.

ANS: A Myelomeningocele is one of the most common causes of neuropathic (neurogenic) bladder dysfunction among children. Risk of intellectual impairment is minimized through early intervention and management of hydrocephalus. Respiratory compromise is not a common problem in myelomeningocele. Cranioschisis is a skull defect through which various tissues protrude. It is not associated with myelomeningocele. p1582

A young boy has just been diagnosed with pseudohypertrophic (Duchenne's) muscular dystrophy. The management plan should include: a. Recommending genetic counseling. b. Explaining that the disease is easily treated. c. Suggesting ways to limit the use of muscles. d. Assisting the family in finding a nursing facility to provide his care.

ANS: A Pseudohypertrophic (Duchenne's) muscular dystrophy is inherited as an X-linked recessive gene. Genetic counseling is recommended for parents, female siblings, maternal aunts, and their female offspring. No effective treatment exists at this time for childhood muscular dystrophy. Maintaining optimal function of all muscles for as long as possible is the primary goal. It has been found that children who remain as active as possible are able to avoid wheelchair confinement for a longer time. Assisting the family in finding a nursing facility is inappropriate at the time of diagnosis. When the child becomes increasingly incapacitated, the family may consider home-based care, a skilled nursing facility, or respite care to provide the necessary care. p1588

The nurse in the neonatal intensive care unit is caring for an infant with myelomeningocele scheduled for surgical repair in the morning. Which early signs of infection should the nurse monitor on this infant (Select all that apply)? a. Temperature instability b. Irritability c. Lethargy d. Bradycardia e. Hypertension

ANS: A, B, C The nurse should observe an infant with unrepaired myelomeningocele for early signs of infection, such as temperature instability (axillary), irritability, and lethargy. Bradycardia and hypertension are not early signs of infection in infants. ch49 Neuromuscular/muscular dysfunction p1583

The nurse is conducting discharge teaching with parents of a preschool child with myelomeningocele, repaired at birth, who is being discharged from the hospital after a urinary tract infection (UTI). Which should the nurse include in the discharge instructions related to management of the child's genitourinary function (Select all that apply)? a. Continue to perform the clean intermittent catheterizations (CIC) at home. b. Administer the oxybutynin chloride (Ditropan) as prescribed. c. Reduce fluid intake in the afternoon and evening hours. d. Monitor for signs of a recurrent UTI. e. Administer furosemide (Lasix) as prescribed.

ANS: A, B, D Discharge teaching to prevent renal complications in a child with myelomeningocele include: (1) regular urologic care with prompt and vigorous treatment of infections; (2) a method of regular emptying of the bladder, such as clean intermittent catheterization (CIC) taught to and performed by parents and self-catheterization taught to children; and (3) medications to improve bladder storage and continence, such as oxybutynin chloride (Ditropan) and tolterodine (Detrol). Fluids should not be limited, and Lasix is not used to improve renal function for children with myelomeningocele. ch49 Neuromuscular/muscular dysfunction p1582

Which assessment findings should the nurse note in a school-age child with Duchenne's muscular dystrophy (DMD) (Select all that apply)? a. Lordosis b. Gower's sign c. Kyphosis d. Scoliosis e. Waddling gait

ANS: A, B, E Difficulties in running, riding a bicycle, and climbing stairs are usually the first symptoms noted in DMD. Typically, affected boys have a waddling gait and lordosis, fall frequently, and develop a characteristic manner of rising from a squatting or sitting position on the floor (Gower's sign). Lordosis occurs as a result of weakened pelvic muscles, and the waddling gait is a result of weakness in the gluteus medius and maximus muscles. Kyphosis and scoliosis are not assessment findings with DMD. ch49 Neuromuscular/muscular dysfunction p1588

A 14-year-old girl is in the intensive care unit after a spinal cord injury 2 days ago. Which nursing care interventions are needed for this child (Select all that apply)? a. Monitoring and maintaining systemic blood pressure. b. Administering corticosteroids. c. Minimizing environmental stimuli. d. Discussing long-term care issues with the family. e. Monitoring for respiratory complications.

ANS: A, B, E Spinal cord injury patients are physiologically labile, and close monitoring of blood pressure and respirations is required. They may be unstable for the first few weeks after the injury. Corticosteroids are administered to minimize the inflammation present with the injury. Minimizing environmental stimuli and discussing long-term care issues with the family do not apply to providing care for this patient. p1596

The nurse is caring for an infant with myelomeningocele scheduled for surgical closure in the morning. Which interventions should the nurse plan for the care of the myelomeningocele sac? a. Open to air b. Covered with a sterile, moist, nonadherent dressing c. Reinforcement of the original dressing if drainage noted d. A diaper secured over the dressing

ANS: B Before surgical closure, the myelomeningocele is prevented from drying by the application of a sterile, moist, nonadherent dressing over the defect. The moistening solution is usually sterile normal saline. Dressings are changed frequently (every 2 to 4 hours), and the sac is closely inspected for leaks, abrasions, irritation, and any signs of infection. The sac must be carefully cleansed if it becomes soiled or contaminated. The original dressing would not be reinforced but changed as needed. A diaper is not placed over the dressing because stool contamination can occur. p1583

The nurse is caring for a neonate born with a myelomeningocele. Surgery to repair the defect is scheduled the next day. The most appropriate way to position and feed this neonate is to place him: a. Prone and tube feed. b. Prone, turn head to side, and nipple feed. c. Supine in infant carrier and nipple feed. d. Supine, with defect supported with rolled blankets, and nipple feed.

ANS: B In the prone position, feeding is a problem. The infant's head is turned to one side for feeding. If the child is able to nipple feed, no indication is present for tube feeding. Before surgery, the infant is kept in the prone position to minimize tension on the sac and risk of trauma. p1583

How much folic acid is recommended for women of childbearing age? a. 0.1 mg b. 0.4 mg c. 1.5 mg d. 2 mg

ANS: B It has been estimated that a daily intake of 0.4 mg of folic acid in women of childbearing age will prevent 50% to 70% of cases of neural tube defects. A dose of 0.1 mg is too low, and 1.5 mg and 2 mg are not recommended dosages of folic acid. p1582

Which finding should cause the nurse to suspect a diagnosis of spastic cerebral palsy? a. Tremulous movements at rest and with activity b. Sudden jerking movement caused by stimuli c. Writhing, uncontrolled, involuntary movements d.Clumsy, uncoordinated movements

ANS: B Spastic cerebral palsy, the most common type of cerebral palsy, will manifest with hypertonicity and increased deep tendon reflexes. The child's muscles are very tight and any stimuli may cause a sudden jerking movement. Tremulous movements are characteristic of rigid/tremor/atonic cerebral palsy. Slow, writhing, uncontrolled, involuntary movements occur with athetoid or dyskinetic cerebral palsy. Clumsy movements, loss of coordination, equilibrium, and kinesthetic sense occur in ataxic cerebral palsy. p1572

The nurse is admitting a school-age child with suspected Guillain-Barré syndrome (GBS). Which nursing intervention is a priority in the care for this child? a. Monitoring intake and output b. Assessing respiratory efforts c. Placing on a telemetry monitor d. Obtaining laboratory studies

ANS: B Treatment of GBS is primarily supportive. In the acute phase, patients are hospitalized because respiratory and pharyngeal involvement may require assisted ventilation, sometimes with a temporary tracheotomy. Treatment modalities include aggressive ventilatory support in the event of respiratory compromise, administration of intravenous immunoglobulin (IVIG), and sometimes steroids; plasmapheresis and immunosuppressive drugs may also be used. Monitoring intake and output, telemetry monitoring, and obtaining laboratory studies may be part of the plan of care but are not the priority. p1591

A home care nurse is caring for an adolescent with a T1 spinal cord injury. The adolescent suddenly becomes flushed, hypertensive, and diaphoretic. Which intervention should the nurse perform first? a. Place the adolescent in a flat right side-lying position. b. Place a cool washcloth on the adolescent's forehead and continue to monitor the blood pressure. c. Implement a standing prescription to empty the bladder with a sterile in-and-out Foley catheter. d. Take a full set of vital signs and notify the health care provider.

ANS: C The adolescent is experiencing an autonomic dysreflexia episode. The paralytic nature of autonomic function is replaced by autonomic dysreflexia, especially when the lesions are above the mid-thoracic level. This autonomic phenomenon is caused by visceral distention or irritation, particularly of the bowel or bladder. Sensory impulses are triggered and travel to the cord lesion, where they are blocked, which causes activation of sympathetic reflex action with disturbed central inhibitory control. Excessive sympathetic activity is manifested by a flushing face, sweating forehead, pupillary constriction, marked hypertension, headache, and bradycardia. The precipitating stimulus may be merely a full bladder or rectum or other internal or external sensory input. It can be a catastrophic event unless the irritation is relieved. Positioning the adolescent, placing a cool washcloth on the adolescent's forehead, continuing to monitor blood pressure and vital signs, and notifying the health care provider would not reverse the sympathetic reflex situation. p1598

A current recommendation to prevent neural tube defects is the supplementation of: a. Vitamin A throughout pregnancy. b. Multivitamin preparations as soon as pregnancy is suspected. c. Folic acid for all women of childbearing age. d. Folic acid during the first and second trimesters of pregnancy.

ANS: C The widespread use of folic acid among women of childbearing age is expected to decrease the incidence of spina bifida significantly. Vitamin A and multivitamin preparations do not have a relation to the prevention of spina bifida. Folic acid supplementation is recommended for the preconceptual period and during the pregnancy. Only 42% of women actually follow these guidelines. p1582

The nurse is admitting a child with Werdnig-Hoffmann disease (spinal muscular atrophy type 1). Which signs and symptoms are associated with this disease? a. Spinal muscular atrophy b. Neural atrophy of muscles c. Progressive weakness and wasting of skeletal muscle d. Pseudohypertrophy of certain muscle groups

ANS: C Werdnig-Hoffmann disease (spinal muscular atrophy type 1) is the most common paralytic form of floppy infant syndrome (congenital hypotonia). It is characterized by progressive weakness and wasting of skeletal muscle caused by degeneration of anterior horn cells. Kugelberg-Welander syndrome is a juvenile spinal muscular atrophy with a later onset. Charcot-Marie-Tooth disease is a form of progressive neural atrophy of muscles supplied by the peroneal nerves. Progressive weakness of the distal muscles of the arms and feet is found. Duchenne's muscular dystrophy is characterized by muscles, especially in the calves, thighs, and upper arms, that become enlarged from fatty infiltration and feel unusually firm or woody on palpation. The term pseudohypertrophy is derived from this muscular enlargement. p1585

The nurse is preparing to admit a newborn with myelomeningocele to the neonatal intensive care nursery. Which describes this newborn's defect? a. Fissure in the spinal column that leaves the meninges and the spinal cord exposed b. Herniation of the brain and meninges through a defect in the skull c. Hernial protrusion of a saclike cyst of meninges with spinal fluid but no neural elements d. Visible defect with an external saclike protrusion containing meninges, spinal fluid, and nerves

ANS: D A myelomeningocele is a visible defect with an external saclike protrusion, containing meninges, spinal fluid, and nerves. Rachischisis is a fissure in the spinal column that leaves the meninges and the spinal cord exposed. Encephalocele is a herniation of brain and meninges through a defect in the skull, producing a fluid-filled sac. Meningocele is a hernial protrusion of a saclike cyst of meninges with spinal fluid, but no neural elements. p1580

Therapeutic management of a child with tetanus includes the administration of: a. Nonsteroidal antiinflammatory drugs (NSAIDs) to reduce inflammation. b. Muscle stimulants to counteract muscle weakness. c. Bronchodilators to prevent respiratory complications. d. Antibiotics to control bacterial proliferation at the site of injury.

ANS: D Antibiotics are administered to control the proliferation of the vegetative forms of the organism at the site of infection. Tetanus toxin acts at the myoneural junction to produce muscular stiffness and lowers the threshold for reflex excitability. NSAIDs are not routinely used. Sedatives or muscle relaxants are used to help reduce titanic spasm and prevent seizures. Respiratory status is carefully evaluated for any signs of distress because muscle relaxants, opioids, and sedatives that may be prescribed may cause respiratory depression. Bronchodilators would not be used unless specifically indicated. p1592

The parents of a child with cerebral palsy ask the nurse if any drugs can decrease their child's spasticity. The nurse's response should be based on knowing that: a. Anticonvulsant medications are sometimes useful for controlling spasticity. b. Medications that would be useful in reducing spasticity are too toxic for use with children. c. Many different medications can be highly effective in controlling spasticity. d. Implantation of a pump to deliver medication into the intrathecal space to decrease spasticity has recently become available.

ANS: D Baclofen given intrathecally is best suited for children with severe spasticity that interferes with activities of daily living and ambulation. Anticonvulsant medications are used when seizures occur in children with cerebral palsy. The intrathecal route decreases the side effects of the drugs that reduce spasticity. Few medications are presently available for the control of spasticity. ch49 Neuromuscular/muscular dysfunction p1574

What is the most appropriate nursing response to the father of a newborn infant with myelomeningocele who asks about the cause of this condition? a. "One of the parents carries a defective gene that causes myelomeningocele." b. "A deficiency in folic acid in the father is the most likely cause." c. "Offspring of parents who have a spinal abnormality are at greater risk for myelomeningocele." d. "There may be no definitive cause identified."

ANS: D The etiology of most neural tube defects is unknown in most cases. There may be a genetic predisposition or a viral origin, and the disorder has been linked to maternal folic acid deficiency; however, the actual cause has not been determined. There is no evidence that children who have parents with spinal problems are at greater risk for neural tube defects. p1581

The nurse is caring for an intubated infant with botulism in the pediatric intensive care unit. Which health care provider prescriptions should the nurse clarify with the health care provider before implementing? a. Administer 250 mg botulism immune globulin intravenously (BIG-IV) one time. b. Provide total parenteral nutrition (TPN) at 25 mL/hr intravenously. c. Titrate oxygen to keep pulse oximetry saturations greater than 92. d. Administer gentamicin sulfate (Garamycin) 10 mg per intravenous piggyback every 12 hours.

ANS: D The nurse should clarify the administration of an aminoglycoside antibiotic. Antibiotic therapy is not part of the management of infant botulism because the botulinum toxin is an intracellular molecule, and antibiotics would not be effective; aminoglycosides in particular should not be administered because they may potentiate the blocking effects of the neurotoxin. Treatment consists of immediate administration of botulism immune globulin intravenously (BIG-IV) without delaying for laboratory diagnosis. Early administration of BIG-IV neutralizes the toxin and stops the progression of the disease. The human-derived botulism antitoxin (BIG-IV) has been evaluated and is now available nationwide for use only in infant botulism. Approximately 50% of affected infants require intubation and mechanical ventilation; therefore, respiratory support is crucial, as is nutritional support because these infants are unable to feed. p1594

The nurse is talking to a parent with a child who has a latex allergy. Which statement by the parent would indicate a correct understanding of the teaching? a. "My child will have an allergic reaction if he comes in contact with yeast products." b. "My child may have an upset stomach if he eats a food made with wheat or barley." c. "My child will probably develop an allergy to peanuts." d. "My child should not eat bananas or kiwis."

ANS: D There are cross-reactions between latex allergies and a number of foods such as bananas, avocados, kiwi, and chestnuts. Although yeast products, wheat and barley, and peanuts are potential allergens, they are currently not known to cross-react with latex. p1585

A 3-month-old infant has been admitted with a diagnosis of encephalitis. The first nursing priority would be to assess:

Ability to maintain airway

The nurse places a young child scheduled for a lumbar puncture in a side-lying position with the head flexed and knees drawn up to the chest. The mother asks why the child has to be positioned this way. The nurse explains the rationale for the positioning is that:

Access to the spinal fluid is facilitated

All of the following are preparations the nurse should do to prepare the room for an admission of a child with seizures except?

Administer Tylenol for comfort

A school-age child with leukemia experienced severe nausea and vomiting when receiving chemotherapy for the first time. Which is the most appropriate nursing action to prevent or minimize these reactions with subsequent treatments?

Administer an antiemetic before chemotherapy begins.

The nurse is admitting a young child to the hospital because bacterial meningitis is suspected. Which of the following is the major priority of nursing care?

Administer antibiotic therapy as soon as it is ordered

A child with leukemia has developed pancytopenia. The nurse would institute which measures designed to reduce stomatitis in this child during the course of chemotherapy? (Select all that apply.)

Administering pain medication as ordered Rinsing the mouth several times a day with plain water

At what developmental period do children have the most difficulty coping with death particularly if it is their own?

Adolescence

A nurse is caring for a school-age child who has suspected osteosarcoma. Which of the following laboratory values is likely to be elevated if this diagnosis is correct?

Alkaline phosphatase

A 3-year-old child is admitted to the hospital unit with a diagnosis of viral meningitis. The nurse should take which of the following actions in the care of this child? (Select all that apply)

Allow the child to assume a position of comfort, Administer acetaminophen for pain, Monitor the child for seizures

A 3-year-old child is hospitalized after a near-drowning accident. The child's mother complains to the nurse, "This seems unnecessary when he is perfectly fine." The nurse's best reply is: complications could still occur."

Although many children do not appear to have suffered adverse effects from the event, complications such as respiratory compromise and cerebral edema may occur up to 24 hours after the incident

The nurse should recommend medical attention if a child with a slight head injury experiences: Confusion or abnormal behavior.

An adolescent boy is brought to the emergency department after a motorcycle accident. His respirations are deep, periodic, and gasping. There are extreme fluctuations in blood pressure.

is an ominous sign of brainstem especially medullary dysfunction that often precedes complete apnea.

An appropriate nursing intervention when caring for an unconscious child should be to: Monitor fluid intake and output carefully to avoid fluid overload and cerebral edema.

Several nurses tell their nursing supervisor that they want to be able to attend the funeral of a child for whom they had cared. They say they felt especially close to both the child and the family. The supervisor should recognize that attending the funeral is:

Appropriate because it can assist in the resolution of personal grief.

A child has been treated with chemotherapy for cancer. The nurse anticipates that neutropenia is an expected consequence and teaches the parents to:

Avoid crowded spaces

A 3-year old child with meningitis is documented as having photophobia. Which of the following interventions is most effective in minimizing this symptom?

Avoid using the TV

An advantage to using a fiberglass cast instead of a plaster cast is that a fiberglass cast: a. Is less expensive. c. Molds closely to body parts. b. Dries rapidly. d. Has a smooth exterior.

B A synthetic casting material dries in 5 to 30 minutes as compared with a plaster cast, which takes 10 to 72 hours to dry. Synthetic casts are more expensive. Plaster casts mold closer to body parts. Synthetic casts have a rough exterior, which may scratch surfaces.

Kristin, age 10 years, sustained a fracture in the epiphyseal plate of her right fibula when she fell off of a tree. When discussing this injury with her parents, the nurse should consider which statement? a. Healing is usually delayed in this type of fracture. b. Growth can be affected by this type of fracture. c. This is an unusual fracture site in young children. d. This type of fracture is inconsistent with a fall.

B Detection of epiphyseal injuries is sometimes difficult, but fractures involving the epiphysis or epiphyseal plate present special problems in determining whether bone growth will be affected. Healing of epiphyseal injuries is usually prompt. The epiphysis is the weakest point of the long bones. This is a frequent site of damage during trauma.

The nurse is caring for a 4-year-old child immobilized by a fractured hip. Which complications should the nurse monitor? a. Hypocalcemia b. Decreased metabolic rate c. Positive nitrogen balance d. Increased production of stress hormones

B Immobilization causes a decreased metabolic rate with slowing of all systems and a decreased food intake, leads to hypercalcemia, and causes a negative nitrogen balance secondary to muscle atrophy. A decreased production of stress hormones occurs with decreased physical and emotional coping capacity.

Which interaction is part of the discharge plan for a school-age child with osteomyelitis who is receiving home antibiotic therapy? a. Instructions for a low-calorie diet b. Arrangements for tutoring and schoolwork c. Instructions for a high-fat, low-protein diet d. Instructions for the parent to return the child to team sports immediately

B Promoting optimal growth and development in the school-age child is important. It is important to continue schoolwork and arrange for tutoring if indicated. The child with osteomyelitis should be on a high-calorie, high-protein diet. The child with osteomyelitis may need time for the bone to heal before returning to full activities.

An appropriate nursing intervention when caring for a child in traction is to: a. Remove adhesive traction straps daily to prevent skin breakdown. b. Assess for tightness, weakness, or contractures in uninvolved joints and muscles. c. Provide active range-of-motion exercises to affected extremity 3 times a day. d. Keep child in one position to maintain good alignment.

B Traction places stress on the affected bone, joint, and muscles. The nurse must assess for tightness, weakness, or contractures developing in the uninvolved joints and muscles. The adhesive straps should be released/replaced only when absolutely necessary. Active, passive, or active with resistance exercises should be carried out for the unaffected extremity only. Movement is expected with children. Each time the child moves, the nurse should check to ensure that proper alignment is maintained.

A clinic nurse is conducting a staff in-service for other clinic staff regarding the signs and symptoms of a rhabdomyosarcoma tumor. Which should be included in the teaching session (Select all that apply)? a. Bone fractures b. Abdominal mass c. Sore throat and ear pain d. Headache e. Ecchymosis of conjunctiva

B, C, E The initial signs and symptoms of rhabdomyosarcoma tumors are related to the site of the tumor and compression of adjacent organs. Some tumor locations, such as the orbit, manifest early in the course of the illness. Other tumors, such as those of the retroperitoneal area, only produce symptoms when they are relatively large and compress adjacent organs. Unfortunately, many of the signs and symptoms attributable to rhabdomyosarcoma are vague and frequently suggest a common childhood illness, such as "earache" or "runny nose." An abdominal mass, sore throat and ear pain, and ecchymosis of conjunctiva are signs of a rhabdomyosarcoma tumor. Bone fractures would be seen in osteosarcoma and a headache is a sign of a brain tumor.

The nurse is providing support to a family who is experiencing anticipatory grief related to their child's imminent death. Which of the following is an appropriate nursing intervention?

Be available to family.

When performing the initial assessment, the nurse found that when the infant's head was flexed his knees and hips also flexed. The nurse should document this finding as a

Brudzinski's sign

A school-age child with spina bifida has a shunt in place for hydrocephalus. Which of the following signs and symptoms is not a manifestation of increased intracranial pressure?

Bulging anterior fontanel

Which statement is accurate concerning a child's musculoskeletal system and how it may be different from an adult's? a. Growth occurs in children as a result of an increase in the number of muscle fibers. b. Infants are at greater risk for fractures because their epiphyseal plates are not fused. c. Because soft tissues are resilient in children, dislocations and sprains are less common than in adults. d. Children's bones have less blood flow.

C Because soft tissues are resilient in children, dislocations and sprains are less common than in adults. A child's growth occurs because of an increase in size rather than an increase in the number of the muscle fibers. Fractures in children younger than 1 year are unusual because a large amount of force is necessary to fracture their bones. A child's bones have greater blood flow than an adult's bones.

When infants are seen for fractures, which nursing intervention is a priority? a. No intervention is necessary. It is not uncommon for infants to fracture bones. b. Assess the family's safety practices. Fractures in infants usually result from falls. c. Assess for child abuse. Fractures in infants are often nonaccidental. d. Assess for genetic factors.

C Fractures in infants warrant further investigation to rule out child abuse. Fractures in children younger than 1 year are unusual because of the cartilaginous quality of the skeleton; a large amount of force is necessary to fracture their bones. Infants should be cared for in a safe environment and should not be falling. Fractures in infancy are usually nonaccidental rather than related to a genetic factor.

When does idiopathic scoliosis become most noticeable? a. Newborn period c. During preadolescent growth spurt b. When child starts to walk d. Adolescence

C Idiopathic scoliosis is most noticeable during the preadolescent growth spurt and is seldom apparent before age 10 years.

The nurse is teaching a family how to care for their infant in a Pavlik harness to treat developmental dysplasia of the hip. What should be included? a. Apply lotion or powder to minimize skin irritation. b. Remove the harness several times a day to prevent contractures. c. Return to the clinic every 1 to 2 weeks. d. Place a diaper over harness, preferably using a superabsorbent disposable diaper that is relatively thin.

C Infants have a rapid growth pattern. The child needs to be assessed by the practitioner every 1 to 2 weeks for possible adjustments. Lotions and powders should not be used with the harness. The harness should not be removed, except as directed by the practitioner. A thin disposable diaper can be placed under the harness.

The primary method of treating osteomyelitis is: a. Joint replacement. c. Intravenous antibiotic therapy. b. Bracing and casting. d. Long-term corticosteroid therapy.

C Osteomyelitis is an infection of the bone, most commonly caused by Staphylococcus aureus. The treatment of choice is antibiotics. Joint replacement, bracing and casting, and long-term corticosteroids are not indicated for infectious processes.

Which type of traction uses skin traction on the lower leg and a padded sling under the knee? a. Dunlop c. Russell b. Bryant's d. Buck's extension

C Russell traction uses skin traction on the lower leg and a padded sling under the knee. The combination of longitudinal and perpendicular traction allows realignment of the lower extremity and immobilizes the hips and knees in a flexed position. Dunlop traction is an upper extremity traction used for fractures of the humerus. Bryant's traction is skin traction with the legs flexed at a 90-degree angle at the hip. Buck's extension traction is a type of skin traction with the legs in an extended position. It is used primarily for short-term immobilization, before surgery with dislocated hips, for correcting contractures, or for bone deformities such as Legg-Calvé-Perthes disease.

A neonate is born with mild clubfeet. When the parents ask the nurse how this will be corrected, the nurse should explain that: a. Traction is tried first. b. Surgical intervention is needed. c. Frequent, serial casting is tried first. d. Children outgrow this condition when they learn to walk.

C Serial casting, the preferred treatment, is begun shortly after birth before discharge from the nursery. Successive casts allows for gradual stretching of skin and tight structures on the medial side of the foot. Manipulation and casting of the leg are repeated frequently (every week) to accommodate the rapid growth of early infancy. Surgical intervention is done only if serial casting is not successful. Children do not improve without intervention.

An important nursing consideration when caring for a child with juvenile idiopathic arthritis (JIA) is to: a. Apply ice packs to relieve stiffness and pain. b. Administer acetaminophen to reduce inflammation. c. Teach child and family the correct administration of medications. d. Encourage range-of-motion exercises during periods of inflammation.

C The management of JIA is primarily pharmacologic. The family should be instructed regarding administration of medications and the value of a regular schedule of administration to maintain a satisfactory blood level in the body. They need to know that nonsteroidal antiinflammatory drugs should not be given on an empty stomach and to be alert for signs of toxicity. Warm, moist heat is best for relieving stiffness and pain. Acetaminophen does not have antiinflammatory effects. Range-of-motion exercises should not be done during periods of inflammation.

A nurse is conducting discharge teaching for parents of an infant with osteogenesis imperfecta (OI). Further teaching is indicated if the parents make which statement? a. "We will be very careful handling the baby." b. "We will lift the baby by the buttocks when diapering." c. "We're glad there is a cure for this disorder." d. "We will schedule follow-up appointments as instructed."

C The treatment for OI is primarily supportive. Although patients and families are optimistic about new research advances, there is no cure. The use of bisphosphonate therapy with IV pamidronate to promote increased bone density and prevent fractures has become standard therapy for many children with OI; however, long bones are weakened by prolonged treatment. Infants and children with this disorder require careful handling to prevent fractures. They must be supported when they are being turned, positioned, moved, and held. Even changing a diaper may cause a fracture in severely affected infants. These children should never be held by the ankles when being diapered but should be gently lifted by the buttocks or supported with pillows. Follow-up appointments for treatment with bisphosphonate can be expected.

The nurse is caring for a preschool child with a cast applied recently for a fractured tibia. Which assessment findings indicate possible compartment syndrome (Select all that apply)? a. Palpable distal pulse b. Capillary refill to extremity of <3 seconds c. Severe pain not relieved by analgesics d. Tingling of extremity e. Inability to move extremity

C, D, E Indications of compartment syndrome are severe pain not relieved by analgesics, tingling of extremity, and inability to move extremity. A palpable distal pulse and capillary refill to the extremity of <3 seconds are expected findings.

The nurse is caring for a child dying from cancer. Physical signs that the child is approaching death include which of the following?

Change in respiratory pattern

A child is to receive chemotherapy with a vesicant drug by the IV route. The nurse can ensure safe administration of this drug by doing which of the following?

Checking for blood return before, during, and after administration of the drug

A child diagnosed with Ewing's sarcoma is being treated with chemotherapy. The results of a complete blood count (CBC) indicate severe thrombocytopenia. Nursing interventions related to this finding would include: (Select all that apply.)

Clearing the floor of the child's room to prevent falls and bruises, Minimizing needle sticks and intrusive procedures

treatment of brain tumors in children consists of which therapies Surgery Chemotherapy Radiation Treatment for brain tumors in children may consist of surgery, chemotherapy, and radiotherapy alone or in combination

Clinical manifestations of increased intracranial pressure (ICP) in infants are Diplopia and blurred vision. Irritability.Distended scalp veins.

Which nursing intervention is appropriate to assess for neurovascular competency in a child who fell off the monkey bars at school and hurt his arm? a. The degree of motion and ability to position the extremity. b. The length, diameter, and shape of the extremity. c. The amount of swelling noted in the extremity and pain intensity. d. The skin color, temperature, movement, sensation, and capillary refill of the extremity.

D A neurovascular evaluation includes assessing skin color and temperature, ability to move the affected extremity, degree of sensation experienced, and speed of capillary refill in the extremity. The degree of motion in the affected extremity and ability to position the extremity are incomplete assessments of neurovascular competency. The length, diameter, and shape of the extremity are not assessment criteria in a neurovascular evaluation. Although the amount of swelling is an important factor in assessing an extremity, it is not a criterion for a neurovascular assessment.

What would cause a nurse to suspect that an infection has developed under a cast? a. Complaint of paresthesia c. Increased respirations b. Cold toes d. "Hot spots" felt on cast surface

D If hot spots are felt on the cast surface, they usually indicate infection beneath the area. This should be reported so a window can be made in the cast to observe the site. The "five Ps" of ischemia from a vascular injury include pain, pallor, pulselessness, paresthesia, and paralysis. Paresthesia is an indication of vascular injury, not infection. Cold toes may be indicative of too tight a cast and need further evaluation. Increased respirations may indicate a respiratory infection or pulmonary emboli. This should be reported, and the child should be evaluated.

Which term is used to describe an abnormally increased convex angulation in the curvature of the thoracic spine? a. Scoliosis c. Lordosis b. Ankylosis d. Kyphosis

D Kyphosis is an abnormally increased convex angulation in the curve of the thoracic spine. Scoliosis is a complex spinal deformity usually involving lateral curvature, spinal rotation causing rib asymmetry, and thoracic hypokyphosis. Ankylosis is the immobility of a joint. Lordosis is an accentuation of the cervical or lumbar curvature beyond physiologic limits.

Which medication is usually tried first when a child is diagnosed with juvenile idiopathic arthritis (JIA)? a. Aspirin b. Corticosteroids c. Cytotoxic drugs such as methotrexate d. Nonsteroidal antiinflammatory drugs (NSAIDs)

D NSAIDs are the first drugs used in JIA. Naproxen, ibuprofen, and tolmetin are approved for use in children. Aspirin, once the drug of choice, has been replaced by the NSAIDs because they have fewer side effects and easier administration schedules. Corticosteroids are used for life-threatening complications, incapacitating arthritis, and uveitis. Methotrexate is a second-line therapy for JIA.

A 4-year-old child is newly diagnosed with Legg-Calvé-Perthes disease. Nursing considerations should include which action? a. Encouraging normal activity for as long as is possible b. Explaining the cause of the disease to the child and family c. Preparing the child and family for long-term, permanent disabilities d. Teaching the family the care and management of the corrective appliance

D The family needs to learn the purpose, function, application, and care of the corrective device and the importance of compliance to achieve the desired outcome. The initial therapy is rest and non-weight bearing, which helps reduce inflammation and restore motion. Legg-Calvé-Perthes is a disease with an unknown etiology. A disturbance of circulation to the femoral capital epiphysis produces an ischemic aseptic necrosis of the femoral head. The disease is self-limiting, but the ultimate outcome of therapy depends on early and efficient therapy and the child's age at onset.

The nurse is teaching the parents of a 7-year-old child who has just had a cast applied for a fractured arm with the wrist and elbow immobilized. Which instructions should be included in the teaching? a. Swelling of the fingers is to be expected for the next 48 hours. b. Immobilize the shoulder to decrease pain in the arm. c. Allow the affected limb to hang down for 1 hour each day. d. Elevate casted arm when resting and when sitting up.

D The injured extremity should be kept elevated while resting and in a sling when upright. This will increase venous return. Swelling of the fingers may indicate neurovascular damage and should be reported immediately. Permanent damage can occur within 6 to 8 hours. Joints above and below the cast on the affected extremity should be moved. The child should not engage in strenuous activity for the first few days. Rest with elevation of the extremity is encouraged.

The nurse uses the palms of the hands when handling a wet cast to: a. Assess dryness of the cast. c. Keep the patient's limb balanced. b. Facilitate easy turning. d. Avoid indenting the cast.

D Wet casts should be handled by the palms of the hands, not the fingers, to prevent creating pressure points. Assessing dryness, facilitating easy turning, or keeping the patient's limb balanced are not reasons for using the palms of the hand rather than the fingers when handling a wet cast.

Which of the following best describes the 4-year-old's concept of death?

Death is temporary.

Lab calls the nurse with results of the CSF sample that was obtained during the lumbar puncture of the child. The nurse would suspect bacterial meningitis with which of the following CSF results?

Decreased glucose, elevated protein

A nurse is assessing the family of a child brought in for severe injuries. Which of the following behaviors by the parents indicates probable abuse?

Delay in seeking treatment for the child's injuries

A 4-year-old is diagnosed with acute lymphocytic leukemia (ALL). Following teaching about the testing and therapy, the nurse evaluates the family's understanding of the problem. The statement by the family that indicates appropriate knowledge would be: "Tests will:

Determine the extent of the malignant process and stage of the leukemia.

A 17-year-old is being admitted for an amputation related to a bone tumor. The nurse is developing a nursing care plan and determines the most appropriate age-related diagnosis is:

Disturbed body image

A 4-year-old child is being evaluated for hydrocephalus. The nurse notes which of the following as an early sign of hydrocephalus in this child?

Early morning headache

The school health nurse has seen a child several times with the same complaints. The school health nurse would suspect a brain tumor after noting the presence of which of the following symptoms that is compatible with this health problem?

Early morning headache and vomiting

The nurse is assigned to the postoperative care of a child with a below-the-knee amputation for osteogenic sarcoma. The nurse plans to include which of the following in nursing care of the child?

Encourage early visits from friends

At the time of a child's death, the nurse tells his mother, "We will miss him so much." The best interpretation of this is that the nurse is doing which of the following?

Expressing personal feelings of loss.

The nurse is caring for a child who has just died. The parents ask to be left alone so that they can rock their child one more time. The nurse should do which of the following?

Grant their request.

Which information should the nurse give to a child who is to have magnetic resonance imaging (MRI) of the brain?"Your head will be restrained during the procedure."

How should the nurse explain positioning for a lumbar puncture to a 5-year-old child? "You will lie on your side and bend your knees so that they touch your chin."

A child is receiving chemotherapy to induce remission in acute leukemia. When considering common side effects of chemotherapy, the nurse would write which of the following as appropriate nursing diagnoses early in the course of therapy? (Select all that apply.)

Impaired oral mucous membranes Imbalanced nutrition: less than body requirements related to nausea and vomiting

A 10-year-old boy receives a blow to his head with a hard baseball and is admitted to the hospital for observation. If the child were to develop an epidural hematoma, he would most like display symptoms:

In the emergency room or soon after arriving on the unit

Probably the most important criterion on which to base the decision to report suspected child abuse is which of the following?

Incompatibility between the history and injury observed

A nurse is assessing a new admission. The 6-month-old infant displays irritability, bulging fontanels, and setting-sun eyes. The nurse would suspect.

Increased intracranial pressure

A 10-year-old client presents with weakness in legs and history of the flu. The medical diagnosis is Guillain-Barre syndrome. It would be imperative for the nurse to inform the physician after observing which of the following?

Increasing hoarseness

A child with myelomeningocele is started on a bowel management plan. The child's mother questions why this is being done. The nurse's response will be based on the understanding that lack of:

Innervation to the anal sphincter predisposes the child to being incontinent

Which of the following is most descriptive of a school-age child's reaction to death?

Is very interested in funerals and burials

A child is being treated for increased intracranial pressure (ICP). Appropriate actions to decrease ICP would include: (Select all that apply).

Keeping head of bed at a 30-degree angle, Providing supplemental oxygen, Administering IV osmotic diuretics as ordered

Which of the following is an appropriate nursing intervention when providing comfort and support for a child when death is imminent?

Limit care to essentials.

A child with a history of a seizure was admitted 2 hours ago. The history indicates fever, chills, and vomiting for the past 24 hours. In report, the nurse is told that the child has a positive Brudzinski's sign. The nurse infers this most likely caused by:

Meningeal irritation

A 2-year-old is admitted to the neurosurgical unit following a head injury. The nurse is using the Glasgow Coma Scale to measure neurological functioning. Which of the following assessment findings indicate the lowest level of functioning for this child?

No response to painful stimuli

The nurse is caring for a toddler who has had surgery for a brain tumor. During an assessment the nurse notes that the child is becoming irritable and pupils are unequal and sluggish. The most appropriate nursing action is to:

Notify the practitioner immediately

A 6-month old infant is admitted to the PICU with a basilar skull fracture. Which of the following assessment findings would the nurse not expect?

Nuchal rigidity

An 8-year-old client with a ventriculoperitoneal shunt was admitted for shunt malfunction. He presents with symptoms of increased intracranial pressure. The mechanisms of the development of his symptoms is most probably related to:

Obstructed flow of cerebrospinal fluid

A 15-year-old client is seen in the emergency department following a head injury from football. During the first few hours after admission, he sleeps unless awakened, but he can be aroused easily and is oriented. In charting assessment findings, the nurse would describe this level of consciousness as:

Obtunded

The nurse observes a client with the neck and back arched and extremities severely extended. The mother asks why the child is doing that. The nurse explains that this posturing is called:

Opisthotonos

The nurse is providing client education for a family whose child has cerebral palsy and is receiving baclofen epidural therapy to control spasticity. Which of the following is most important for the nurse to include in the discussion?

Parents must bring the child back to the clinic on a regular basis to have more medicine added to the pump

A nurse is evaluating a family in which child abuse has occurred. Which of the following best indicates that the parents are making progress toward resolution of the violence?

Parents relate an understanding of normal growth and development

Focal seizures may arise from any area of the cerebral cortex, but the frontal, temporal, and parietal lobes are most commonly affected. P

Partial seizures are caused by abnormal electrical discharges from epileptogenic foci limited to a circumscribed region of the cerebral cortex

Upon performing a physical assessment of a 7-month-old child, the nurse notes the following findings. The nurse concludes that which finding is abnormal and could suggest cerebral palsy (CP)?

Positive tonic neck reflex

Which of the following best describes how preschoolers react to the death of a loved one?

Preschooler may feel guilty and responsible for the death.

A common characteristic of those who sexually abuse children is which of the following?

Pressure victim into secrecy

An 18-month-old child is observed having a seizure. The nurse notes that the child's jaws are clamped. The priority nursing responsibility at this time would be:

Protect the child from harm from the environment

The parents of a child with neutropenia secondary to chemotherapy have been taught protective isolation behaviors. Nursing observations that indicate a need for further education is when the parents:

Pull the child in a wagon around the nursing unit for entertainment

The nurse is doing a neurologic assessment on a child whose level of consciousness has been variable since sustaining a cervical neck injury 12 hours ago. Which of the following is the priority assessment for this child?

Reactivity of pupils

A preschooler is found digging up a pet bird that was recently buried after it died. The best explanation for this behavior is which of the following?

Reassurance is needed that the pet has not gone somewhere else.

A child is admitted with possible Reye syndrome. Which of the following factors in the child's health history might be supportive of this diagnosis?

Recent episode of gastroenteritis

An adolescent receiving cyclophosphamide (Cytoxan) for acute lymphocytic leukemia (ALL) asks the nurse to come quickly to evaluate "blood in my urine." The most important action by the nurse would be to:

Recognize that this is untoward and report the event

The nurse is caring for a child with multiple injuries who is comatose. The nurse should recognize that pain:

Requires astute nursing assessment and management

Which of the following is a clinical manifestation of increased ICP in infants?

Shrill, high-pitched cry

The nurse is discussing long-term care with the parents of a child who has a ventriculoperitoneal shunt to correct hydrocephalus. Which of the following should the nurse include?

Shunt malfunction or infection requires immediate treatment

A nurse is caring for a 14-year-old adolescent boy who has been diagnosed with osteosarcoma of the left proximal tibia. The adolescent is 4 days preoperative for an above-the-knee amputation and is admitted to the hospital for preoperative chemotherapy. The adolescent knows about the surgery. Which of the following is the nurse's responsibility in the next 4 days regarding this adolescent's emotional health?

Spend time with the adolescent to answer any questions he may have.

The nurse is administering an IV chemotherapeutic agent to a child with leukemia. The child suddenly begins to wheeze and have severe urticaria. Which is the most appropriate nursing action?

Stop drug infusion immediately.

Which of the following is characteristic of most neonatal seizures?

Subtle and barely discernible

Which of the following positions is the most effective in preventing a further increase in pressure for a child with increased intracranial pressure?

Supine with 30° head elevation

Which of the following sets of vital signs is indicative of Cushing's reflex in a 6-year-old child?

Temperature 37.8°C, pulse 64/min, respiratory rate 20/min, and blood pressure 140/40 mm Hg

The nurse notes that the parents of a critically ill child spend a large amount of time talking with the parents of another child who is also seriously ill. They talk with these parents more than with the nurses. The nurse should recognize that this behavior indicates:

That parent-to-parent support is valuable.

A child has just been diagnosed with bacterial meningitis. The parent asks the nurse how long the child will be in isolation. The nurse's reply will be based on a protocol that isolation continues until:

The antibiotics have been administered for 24 hours

The nursing diagnosis for a child undergoing chemotherapy for leukemia is "Imbalanced nutrition: less than body requirements related to nausea and anorexia." The nurse would formulate which of the following as an appropriate goal for this client?

The child's caloric intake will be within normal range

A 5-year-old girl's sibling dies from sudden infant death syndrome. The parents are concerned because the girl showed more outward grief when her cat died than for her sibling's death. The nurse should explain that:

The death may be so painful and threatening that the child must deny it for now to protect her psyche.

A cure is no longer possible for a young child with cancer. The nursing staff recognizes that the goal of treatment must shift from cure to palliation. Which of the following is an important consideration at this time?

The family is included in the decision to shift the goals of treatment.

asks why a CT scan is required when she "seems fine." nurse should explain that the toddler: history of the fall, brief loss of consciousness, and vomiting four times necessitate evaluation of a potential brain injury.

The nurse is assessing a child who was just admitted to the hospital for observation after a head injury. The most essential part of the nursing assessment to detect early signs of a worsening condition is:Level of consciousness.

The Glasgow Coma Scale consists of an assessment of:The Glasgow Coma Scale assesses eye opening and verbal and motor responses.

The nurse is closely monitoring a child who is unconscious after a fall and notices that the child suddenly has a fixed and dilated pupil. The nurse should interpret this as: Neurosurgical emergency.

The nurse has received report on four children. Which child should the nurse assess first? A preschool child with a head injury and decreasing level of consciousness

The nurse is performing a Glasgow Coma Scale (GCS) on a school-age child with a head injury. The child opens eyes spontaneously, obeys commands, and is oriented to person, time, and place.

caloric test involves the instillation of ice water into the ear of a comatose child. The caloric test is painful and is never performed on a child who is awake or one who has a ruptured tympanic membrane.

The nurse is preparing a school-age child for a computed tomography (CT) scan to assess cerebral function. When preparing the child for the scan, which statement should the nurse include? "The scan will not hurt."

A child will be undergoing chemotherapy. The nurse discusses the issue of hair loss with the child and family before chemotherapy begins. Later the family asks the nurse why this information was given to the child at this time. The nurse's response will include the information that:

The presence or absence of hair is related to body image. Strategies for handling hair loss should precede the event.

Which neurologic diagnostic test gives a visualized horizontal and vertical cross section of the brain at any axis? Computed tomography (CT) scan

The priority nursing intervention when a child is unconscious after a fall is to: Establish an adequate airway.

A young child has just been diagnosed with spastic cerebral palsy. The nurse is teaching the parents how to meet the dietary needs of their child. The nurse would explain that children with cerebral palsy frequently have special dietary needs or feeding challenges because:

The spasticity of their muscles increases their caloric needs

The nurse is talking with the parents of a child who died 6 months ago. They sometimes still "hear" the child's voice and have trouble sleeping. They describe feeling "empty" and depressed. The nurse should recognize which of the following?

These are normal grief responses

Which of the following statements is correct about young children who report sexual abuse by one of their parents?

They may exhibit various behavioral manifestations

Generalized movements of all extremities which would be which type of seizure?

Tonic-clonic

Which of the following describes a child who is abused by the parent(s)?

Unintentially contributes to the abusing situation

The mother of a 1-month-old infant tells the nurse that she worries that her baby will get meningitis like her oldest son did when he was an infant. The nurse should base her response on knowing that:

Vaccination to prevent Haemophilus influenzae type b meningitis has decreased the frequency of this disease in children.

The vector reservoir for agents causing viral encephalitis in the United States is: Mosquitoes and ticks.

What action may be beneficial in reducing the risk of Reye's syndrome? Avoidance of aspirin and ibuprofen for children with varicella or those suspected of having influenza

When taking the history of a child hospitalized with Reye's syndrome, the nurse should not be surprised that a week ago the child had recovered from: varicella or influenza.

When caring for the child with Reye's syndrome, the priority nursing intervention is to: Monitor intake and output.

An important nursing intervention when caring for a child who is experiencing a seizure is to: Describe and record the seizure activity observed.

Which clinical manifestations would suggest hydrocephalus in a neonate? Bulging fontanel and dilated scalp veins

The most common clinical manifestation of brain tumors in children is: Headaches and vomiting.

Which statement best describes a neuroblastoma? Diagnosis is usually made after metastasis occurs.

Which type of fracture describes traumatic separation of cranial sutures? Diastatic

Which statement best describes a subdural hematoma? bleeding that occurs between the dura and the cerebrum as a result of a rupture of cortical veins that bridge the subdural space

Which drug would be used to treat a child who has increased intracranial pressure resulting from cerebral edema? Mannitol an osmotic diuretic, administered IV, is the drug used most frequently for rapid reduction

Which statement is most descriptive of a concussion? It is a transient, reversible neuronal dysfunction. with instantaneous loss of awareness and responsiveness resulting from trauma to the head.

Which term is used to describe a child's level of consciousness when the child can be aroused with stimulation? Obtundation

Which term is used when a patient remains in a deep sleep, responsive only to vigorous and repeated stimulation? stupor

Pinpoint pupils or fixed, bilateral pupils for more than 5 minutes are indicative of brainstem damage.

Which test is never performed on a child who is awake? Oculovestibular response

The nurse's best response is:"The seizure may or may not mean that your child has epilepsy."

Which type of seizure involves both hemispheres of the brain? Generalized Clinical observations of generalized seizures indicate that the initial involvement is from both hemispheres

The initial clinical manifestation of generalized seizures is: Losing consciousness. Being confused, feeling frightened, and seeing flashing lights are clinical manifestations of a complex partial seizure.

Which type of seizure may be difficult to detect? Absence Absence seizures may go unrecognized because little change occurs in the child's behavior during the seizure

A 3-year-old child is brought to the Emergency Department for treatment of injuries the father stated were obtained when the child fell off of his tricycle. Upon assessment, numerous bruised areas, old and fresh, are noted on the child's back, buttocks, and shoulders. Radiologic examination reveals fractured ribs and a healed fractured humerus. Based on these findings, the nurse's next course of action would be to:

ask the father to provide further details of the incident, obtain a medical history of the child, and then interview the child separately

the score the nurse should record? 15 The GCS consists of a three-part assessment: eye opening, verbal response, and motor response Numeric values of 1 through 5 are assigned to the levels of response in each category.

nurse is caring for a child with severe head trauma after a car accident. Which is an ominous sign that often precedes death? Periodic or irregular breathing

The nurse is caring for a neonate with suspected meningitis. Which clinical manifestations should the nurse prepare to assess if meningitis is confirmed Bulging anterior fontanel Weak cry Poor muscle tone

nurse is monitoring an infant for signs of increased intracranial pressure . late signs of increased in an infant Alteration in pupil size and reactivity Extension or flexion posturing Cheyne-Stokes respirations

A school-age child has sustained a head injury and multiple fractures after being thrown from a horse. The child's level of consciousness is variable. The parents tell the nurse that they think their child is in pain because of

periodic crying and restlessness. The most appropriate nursing action is to: Discuss with practitioner what analgesia can be safely administered.

A 3-month-old infant dies shortly after arrival to the Emergency Department. The infant has subdural and retinal hemorrhages but no external signs of trauma. The nurse should expect:

shaken-baby syndrome

A school-age child is diagnosed with a life-threatening illness. The parents want to protect their child from knowing the seriousness of the illness. The nurse should explain that:

terminally ill children know when they are seriously ill.

child has been seizure-free for 2 years. father asks the nurse how much longer the child will need to take the antiseizure medications. nurse includes which intervention in the response? stepwise approach will be

used to reduce the dosage gradually.A predesigned protocol is used to wean a child gradually off antiseizure medications, usually when the child is seizure-free for 2 years and has a normal electroencephalogram


Conjuntos de estudio relacionados

Chapter 9, chapter 10, PM Chapter 11

View Set

Chapter 13 - Services: The Intangible Product (Smartbook)

View Set

Postoperative Nursing Management (NC1)

View Set

Module 3: Biomolecules and MetabolismWhich of the following is NOT one of the six major elements that make up biological macromolecules? Hydrogen Sulfur Phosphorus Calcium Nitrogen

View Set

Archimedes', Pascal's, and Bernoulli's Principle states...

View Set